A new device uses the global positioning system to determine a cow's location and, when a cow strays outside of its p...

smilde11 on October 29, 2018

PT 75, S1, Q21

I correctly chose B, however can you explain why E would not be correct? Thank you!

Reply
Create a free account to read and take part in forum discussions.

Already have an account? log in

Mehran on October 31, 2018

Hi @smilde11, thanks for your post.

This stimulus presents the equivalent of an argument, the conclusion of which is the device maker's prediction "that ranchers will purchase the device at its current price." This is despite the fact that we are also told in the stimulus that outfitting a herd of cattle with this device is "far more expensive than other means of keeping cattle in their pastures, such as fences." So what gives?

The question stem asks us to strengthen the device maker's prediction. Answer choice (B) does this; if cattle in a herd follow the lead of the same few herd members, then it would make sense to make the investment and outfit all cattle in a herd with the device.

Answer choice (E) is irrelevant to the conclusion you are given, which specifically says "at its current price" - no discounts.

Hope that helps. Please let us know if you have additional questions.